3
$\begingroup$

I came across this answer which claims that not every Perron number admits a primitive non-negative integral matrix representation. This seems to contradict Lind's theorem, which states:

If $\lambda$ is a Perron number, then there is an aperiodic non-negative integral matrix with spectral radius $\lambda$.

How is this possible?

The proposed counterexample is the polynomial: $$p(x)=x^6−4x^4−5x^3+10x^2−12x+6$$

The largest root of the polynomial is the Perron number 2.2508.

$\endgroup$
6
  • $\begingroup$ You are right, I made a mistake in the calculation. The largest root of the polynom is indeed the Perron number 2,2508... However, could you please be a bit more specific about, why you do not see a contradiction? Thank you very much. $\endgroup$ Commented Oct 11 at 13:50
  • $\begingroup$ So in other words: There might be another minimal polynom of higher degree with the same Perron number for which a primitive non-negative integral matrix representation may exist? $\endgroup$ Commented Oct 11 at 22:55
  • $\begingroup$ Not quite. There might be some primitive integral matrix $A$ such that $\rho(A)=\lambda$ and $p(x)$ divides the characteristic polynomial $f(x)$ of $A$. However, if $f\ne p$ (i.e., if $n>\deg p$), then $f$ is not the minimal polynomial of $\lambda$. The result of Lind is all about the relationship between $\lambda$ and $\rho(A)$. It has a side effect on $p$ (as $p$ must divide $f$), but the result is not really about $p$. In contrast, the question that the counterexample addresses is all about $p$. $\endgroup$ Commented Oct 11 at 23:26
  • $\begingroup$ Understood. Thank you. Is there a way to find an upper limit for $deg(f(x))$ for a given $p(x)$? $\endgroup$ Commented Oct 13 at 13:04
  • $\begingroup$ Sorry. I have no idea. As Lind’s paper gives an algorithm to obtain a target matrix, you may see if it is possible to obtain an upper bound for the size of the matrix using that algorithm. $\endgroup$ Commented Oct 13 at 18:50

1 Answer 1

2
$\begingroup$

There is no contradiction, because Lind’s theorem has fewer restrictions on the primitive matrix.

Let $p\in\mathbb Z[x]$ be the minimal polynomial of a Perron number $\lambda$. In the cited counterexample, the question it addresses requires that the primitive integral matrix $A$ is similar to the companion matrix of $p$ (which is $x^6−13x^4−20x^3+x^2−x+2$ in that counterexample). In other words, the question requires not only that $p$ is the characteristic polynomial of $A$, but also that $A$ is non-derogatory. If $A$ is $n\times n$, the question also implicitly requires that $n=\deg p$.

In contrast, Lind’s theorem only requires $\lambda$ to be equal to the spectral radius of $A$. It allows $n>\deg p$. It does not require $p$ to be the characteristic polynomial of $A$ (but by the definitions of $p$ and $A$, $p$ must divide the characteristic polynomial of $A$). Nor does it require $A$ to be non-derogatory.

In fact, on p.289, Lind has mentioned that if $\lambda\approx3.8916$ is the Perron number of $p(x)=t^3+3t^2-15t-46$, his algorithm will generate a matrix $A$ with $n=10>3=\deg p$, and so $p$ is only a factor of, but not identical to, the characteristic polynomial of $A$.

$\endgroup$

You must log in to answer this question.

Start asking to get answers

Find the answer to your question by asking.

Ask question

Explore related questions

See similar questions with these tags.